Wie formatiert man die folgenden beiden Optimierungsprobleme?

Wie formatiert man die folgenden beiden Optimierungsprobleme?

Wie kann ich dieses Format zweier Optimierungsprobleme reproduzieren?Bildbeschreibung hier eingeben

Antwort1

  • Willkommen bei TeX.SE!
  • Bitte helfen Sie uns, Ihnen zu helfen: Geben Sie immer das an, was Sie selbst ausprobieren, am besten in Form eines MWE (Minimal Working Example), einem kleinen, aber vollständigen Dokument, das mit beginnt \documentclass{...}und endet mit\end{document}
  • Lösung:
    • Fügen Sie jedes Optimierungsproblem auf eine eigene Miniseite ein
    • arrayfür die Formatierung der Gleichung ist praktisch, Paket zu verwenden
    • für kürzeren und konsistenten Code schlage ich vor, neue mathematische Operatoren für Max und Min zu definieren
  • Da Sie kein MWE als Ausgangspunkt angeben, verwende ich die @scd-Antwort und habe sie entsprechend geändert:
\documentclass{article}
\usepackage{amsmath}
\DeclareMathOperator{\Max}{Max}
\DeclareMathOperator{\Min}{Min}

%\title{My LaTeX Document}

\begin{document}
%\maketitle

\begin{minipage}{0.45\textwidth}
(P)\quad    $\begin{array}[t]{l @{\qquad}r}
            \Max            & 8 x_1 + 3 x_2 \\
            \text{s.a.:}    &   x_1 - 6 x_2 \geq 2 \\
                            & 5 x_1 + 7 x_2=-4 \\
                            &   x_1 \leq 0 \\
                            &   x_2 \geq 0
            \end{array}$
\end{minipage}
\hfill
\begin{minipage}{0.45\textwidth}
(D)\quad    $\begin{array}[t]{l @{\qquad}r}
            \Min            &  2 w_1 - 4 w_2 \\
            \text{s.a.}     &    w_1 + 5 w_2 \leq 8 \\
                            & -6 w_1 + 7 w_2 \geq 3 \\
                            &    w_1 \leq 0 \\
                            &    w_2 \text{ s.r.}
            \end{array}$
\end{minipage}
\end{document}

Bildbeschreibung hier eingeben

Antwort2

Schauen Sie sich den folgenden Code an:

\documentclass{article}
\usepackage[utf8]{inputenc}
\usepackage{amsmath}

\title{My LaTeX Document}

\begin{document}

\maketitle

\begin{minipage}{0.5\textwidth}
\begin{align*}
 \begin{array}{rr}
\text { (P) } \text{ Max } & 8 x_1+3 x_2 \\
\text { s.a.: } & x_1-6 x_2 \geq 2 \\
& 5 x_1+7 x_2=-4 \\
& x_1 \leq 0 \\
& x_2 \geq 0
\end{array}
\end{align*}
\end{minipage}%
\begin{minipage}{0.5\textwidth}
\begin{align*}
 \begin{array}{rr}
& \text { (D) } \quad \operatorname{Min} \quad 2 w_1-4 w_2 \\
& \text { s.a.: } \quad w_1+5 w_2 \leq 8 \\
& -6 w_1+7 w_2 \geq 3 \\
& w_1 \leq 0 \\
& w_2 \quad \text {s.r.}
 \end{array}
\end{align*}
\end{minipage}


\end{document}

Dies führt zu: Bildbeschreibung hier eingeben

Antwort3

Diese Antwort ist nicht dazu gedacht,reproduzierendas gezeigte Format aber irgendwieverbessernes (obwohl dies sehr subjektiv sein kann).

@Zarko hat Ihnen viele wertvolle Ratschläge gegeben (+1). Ich werde einen selbst definierten Befehl verwenden, der sicherstellt, dass alle diese Probleme auf die gleiche Weise gesetzt werden und die folgenden Argumente verwendet:

  • optionales Argument: Standardmäßig wird verwendet \displaystyle. Lassen Sie das optionale Argument leer, um zu haben \textstyle.
  • Erstes obligatorisches Argument: Minimierung/Maximierung, verwenden Sie also \min/ \max/ \Min/ \Max.
  • Zweites obligatorisches Argument: Zielfunktion.
  • Drittes obligatorisches Argument: Alle Ihre Einschränkungen. Beachten Sie, dass dies in eine alignedUmgebung gepackt ist, die es Ihnen ermöglicht, Ihre Einschränkungen auszurichten. Ergebnis
\documentclass{article}
\usepackage{mathtools,array}
\DeclareMathOperator{\Max}{Max}
\DeclareMathOperator{\Min}{Min}

\newcommand{\minmax}[4][\displaystyle]{
    \left\{%
    \begin{array}{@{}>{#1}r@{\quad}>{#1}l@{}}%
                   #2 & #3 \\
        \text{s.\,a.:} & \begin{aligned}[t] #4 \end{aligned}%
    \end{array}%
    \right.%
}

\begin{document}
    \begin{align}
        \mathrm{(P)} &\minmax{\Max}{8x_1+3x_2}{%
            x_1-6x_2    &\geq 2 \\
            5x_1 + 7x_2 &= -4   \\
            x_1         &\leq 0 \\
            x_2         &\geq 0
        }
        &
        \mathrm{(D)} & \minmax{\Min}{2w_1 - 4w_2}{%
            w_1+5w_2   &\leq 8 \\
            -6w_1+7w_2 &\geq 3 \\
            w_1        &\leq 0 \\
            w_2        &\text{ s.\,r.}
        }
    \end{align}
\end{document}

verwandte Informationen